Kathleen Jackson
Test por , creado hace más de 1 año

Module 1 EoT Exam Revision

1489
6
0
Kathleen Jackson
Creado por Kathleen Jackson hace casi 6 años
Cerrar

2808NRS Human Pathophysiology and Pharmacology 2 (Module 1)

Pregunta 1 de 130

1

A 56-year-old male is diagnosed with coronary heart disease. Which of the following modifiable risk factors would most likely influence development of this disease?

Selecciona una de las siguientes respuestas posibles:

  • cigarette smoking.

  • his age.

  • the geographic location of home.

  • his sex.

Explicación

Pregunta 2 de 130

1

In general, atherosclerosis is triggered by:

Selecciona una de las siguientes respuestas posibles:

  • injured neutrophils.

  • deposited adipose cells.

  • macrophages that engulf LDL.

  • lipid-laden mast cells.

Explicación

Pregunta 3 de 130

1

Plaques consisting of lipids, cells, fibrin, and debris, often with attached thrombi that can obstruct arteries and arterioles, are referred to as:

Selecciona una de las siguientes respuestas posibles:

  • Atheromas

  • Sarcomas

  • Fibromas

  • Prothrombi

Explicación

Pregunta 4 de 130

1

The most common cause of myocardial ischaemia is:

Selecciona una de las siguientes respuestas posibles:

  • venous emboli.

  • atherosclerosis.

  • vasospasm.

  • arterial emboli from heart valve vegetation.

Explicación

Pregunta 5 de 130

1

A 32-year-old female presents with lower leg pain and redness. An ultrasound reveals venous thrombus. Which of the following would be most likely to have caused her condition?

Selecciona una de las siguientes respuestas posibles:

  • atherosclerosis.

  • heart valve damage.

  • bacterial infection.

  • vascular injury and inflammation.

Explicación

Pregunta 6 de 130

1

A 75-year-old obese female presents to her GP complaining of oedema in the lower extremities. Physical exam reveals that she has varicose veins. Which of the following is a most likely cause?

Selecciona una de las siguientes respuestas posibles:

  • ischaemia.

  • extreme exercise.

  • long periods of standing.

  • dyslipidemia.

Explicación

Pregunta 7 de 130

1

A 28-year-old female presents with severe chest pain and shortness of breath. She is diagnosed with pulmonary embolism, which most likely originated from the:

Selecciona una de las siguientes respuestas posibles:

  • deep veins of the leg.

  • left ventricle.

  • systemic arteries.

  • superficial veins of the body.

Explicación

Pregunta 8 de 130

1

A 52-year-old male presents with pooling of blood in the veins of the lower extremities and oedema. He is predisposed to:

Selecciona una de las siguientes respuestas posibles:

  • deep vein thrombosis.

  • skin hyperpigmentation.

  • stasis ulcers.

  • all answers are correct.

Explicación

Pregunta 9 de 130

1

A 65-year-old female presents to the ED complaining of difficulty swallowing and shortness of breath. A CT scan would most likely reveal an aneurysm in the:

Selecciona una de las siguientes respuestas posibles:

  • cerebral vessels.

  • illiac arteries.

  • inferior vena cava.

  • thoracic aorta.

Explicación

Pregunta 10 de 130

1

The build-up of fatty plaques within the wall of arteries is:

Selecciona una de las siguientes respuestas posibles:

  • Ischaemic heart disease

  • Peripheral vascular disease

  • Atherosclerosis

  • Hypertension

Explicación

Pregunta 11 de 130

1

Which of the following types of drugs is NOT typically used to treat hypertension?

Selecciona una de las siguientes respuestas posibles:

  • Thiazide diuretics

  • Nitrates

  • ACE inhibitors

  • Calcium channel blockers

Explicación

Pregunta 12 de 130

1

Hypertension that is idiopathic is known as:

Selecciona una de las siguientes respuestas posibles:

  • Angina

  • Secondary

  • Essential

  • Tertiary

Explicación

Pregunta 13 de 130

1

Risk factors for atherosclerosis include all the following except:

Selecciona una de las siguientes respuestas posibles:

  • Low levels of LDL blood serum

  • Hypertension

  • Smoking

  • Advanced age

Explicación

Pregunta 14 de 130

1

A stationary blood clot within an artery is a:

Selecciona una de las siguientes respuestas posibles:

  • Myocardial infarction

  • Embolus

  • Stroke

  • Thrombus

Explicación

Pregunta 15 de 130

1

A 62-year-old male presents to his GP complaining of chest pain at rest and on exertion. He does not have a history of coronary artery disease and reports that the pain often occurs at night. He is most likely experiencing which type of angina?

Selecciona una de las siguientes respuestas posibles:

  • Exertional

  • Stable

  • Variant

  • Unstable

Explicación

Pregunta 16 de 130

1

A 53-year-old male presents with recurrent chest pain on exertion. He is diagnosed with angina pectoris. The pain he experiences occurs when:

Selecciona una de las siguientes respuestas posibles:

  • the vagus nerve is stimulated.

  • myocardial stretch has exceeded the upper limits.

  • the myocardial oxygen supply has fallen below demand.

  • cardiac output has fallen below normal levels.

Explicación

Pregunta 17 de 130

1

A 51-year-old male is at his health clinic for an annual physical exam. After walking from the car to the clinic, he developed a substernal pain. He also reported discomfort in his left shoulder and his jaw, lasting two to three minutes and then subsiding with rest. He indicates that this has occurred frequently over the past few months with similar exertion. He is most likely to be experiencing:

Selecciona una de las siguientes respuestas posibles:

  • stable angina

  • unstable angina

  • variant angina

  • myocardial infarction

Explicación

Pregunta 18 de 130

1

A 52-year-old female is diagnosed with coronary heart disease. She would be expected primarily to suffer from myocardial:

Selecciona una de las siguientes respuestas posibles:

  • hypertrophy.

  • ischaemia.

  • necrosis.

  • dilation.

Explicación

Pregunta 19 de 130

1

Cardiac cells can withstand ischaemic conditions for _____ before irreversible cell injury occurs.

Selecciona una de las siguientes respuestas posibles:

  • 30 minutes

  • 60 minutes

  • 1 minute

  • 20 minutes

Explicación

Pregunta 20 de 130

1

A 49-year-old male presents to his GP complaining of chest pain. An ECG reveals ST elevation. He is diagnosed with myocardial ischaemia. Which of the following interventions would be most beneficial?

Selecciona una de las siguientes respuestas posibles:

  • decrease in ventricular volume with diuretic

  • increase cardiac contractility

  • increase heart rate

  • increase myocardial oxygen supply

Explicación

Pregunta 21 de 130

1

A 55-year-old male died of myocardial infarction. Autopsy would most likely reveal:

Selecciona una de las siguientes respuestas posibles:

  • platelet aggregation within an atherosclerotic coronary artery.

  • decreased ventricular diastolic filling time.

  • embolisation of plaque from the aorta.

  • smooth muscle dysplasia in the coronary artery.

Explicación

Pregunta 22 de 130

1

A 75-year-old male presents with severe chest pain. Lab tests in hospital reveal elevated levels of creatine kinase and lactic dehydrogenase. These elevated levels indicate probable:

Selecciona una de las siguientes respuestas posibles:

  • Unstable angina

  • Hypertension

  • Stable angina

  • Myocardial infarction

Explicación

Pregunta 23 de 130

1

A 60-year-old female had a myocardial infarction. She was brought to hospital 30 minutes later. She survived but now has impaired ventricular function because:

Selecciona una de las siguientes respuestas posibles:

  • there was a temporary alteration in electrolyte balance.

  • the resulting ischaemia led to hypoxic injury and myocardial cell death.

  • there was too much stress on the heart.

  • the cells became hypertrophic.

Explicación

Pregunta 24 de 130

1

Low-density lipoproteins (LDL):

Selecciona una de las siguientes respuestas posibles:

  • transport cholesterol from cells to the liver for excretion.

  • contain only small amounts of cholesterol.

  • promote atheroma development.

  • are associated with low intake of saturated fats.

Explicación

Pregunta 25 de 130

1

Which of the following patients will likely experience difficulty in maintaining lipoprotein synthesis resulting in elevated LDL levels?

Selecciona una de las siguientes respuestas posibles:

  • A 35-year-old patient with a history of hepatitis C and B with end-stage liver disease

  • A 44-year-old female admitted for hysterectomy due to cervical cancer with metastasis

  • A 27-year-old patient with pancreatitis related to alcohol abuse

  • A 55-year-old male admitted for exacerbation of chronic obstructive pulmonary disease (COPD)

Explicación

Pregunta 26 de 130

1

Hypertension can be classified into the following three major categories:

Selecciona una de las siguientes respuestas posibles:

  • Active, passive, fatal

  • Primary, secondary, tertiary

  • Subacute, acute, chronic

  • Essential, secondary, malignant

Explicación

Pregunta 27 de 130

1

Where do atherosclerotic plaques occur most often?

Selecciona una de las siguientes respuestas posibles:

  • in large, abdominal veins

  • at points of arterial vessel bifurcation

  • in myocardial capillaries

  • all answers are correct

Explicación

Pregunta 28 de 130

1

Calcium-channel blocking drugs are effective in:

Selecciona una de las siguientes respuestas posibles:

  • decreasing all types of cardiac arrhythmias.

  • reducing the risk of blood clotting.

  • decreasing the attraction of cholesterol into lipid plaques.

  • reducing cardiac and smooth muscle contractions.

Explicación

Pregunta 29 de 130

1

Which of the following confirms the presence of a myocardial infarction?

Selecciona una de las siguientes respuestas posibles:

  • The presence of elevated serum cardiac enzymes and triglycerides

  • Serum cardiac enzymes markers released from necrotic cells and ECG changes

  • Leukocytosis and elevated C-reactive protein

  • A full description of the pain, including the sequence of development

Explicación

Pregunta 30 de 130

1

High levels of which of the following lipoproteins can be beneficial?

Selecciona una de las siguientes respuestas posibles:

  • IDL

  • LDL

  • HDL

  • VLDL

Explicación

Pregunta 31 de 130

1

A 52-year-old male is diagnosed with primary hypertension. He could be treated with a drug that acts by which of the following mechanisms?

Selecciona una de las siguientes respuestas posibles:

  • calcium channel agonist

  • diuretic

  • angiotensin ll agonist

  • beta-adrenergic agonist

Explicación

Pregunta 32 de 130

1

Claudication describes, or is related to:

Selecciona una de las siguientes respuestas posibles:

  • Painful cramps in skeletal muscles due to peripheral vascular disease

  • Painful upper limbs due to hypertension

  • Chest pain

  • All answers are correct

Explicación

Pregunta 33 de 130

1

The basic pathophysiology of myocardial infarction is best described as:

Selecciona una de las siguientes respuestas posibles:

  • total obstruction of a coronary artery, which causes myocardial necrosis.

  • irregular heart rate and force, reducing blood supply to coronary arteries.

  • temporary vasospasm that occurs in a coronary artery.

  • cardiac output that is insufficient to meet the needs of the heart and body.

Explicación

Pregunta 34 de 130

1

Factors that may precipitate an angina attack include all of the following EXCEPT:

Selecciona una de las siguientes respuestas posibles:

  • eating a large meal.

  • engaging in an angry argument.

  • running upstairs on a hot day.

  • taking a nap.

Explicación

Pregunta 35 de 130

1

Typical early signs or symptoms of myocardial infarction include:

Selecciona una de las siguientes respuestas posibles:

  • persistent chest pain radiating to the left arm, pallor, and rapid, weak pulse.

  • brief, substernal pain radiating to the right arm, with labored breathing.

  • bradycardia, increased blood pressure, and severe dyspnoea.

  • flushed face, rapid respirations, left-side weakness, and numbness.

Explicación

Pregunta 36 de 130

1

Four patients were admitted to the emergency department with severe chest pain. All were given preliminary treatment with aspirin, morphine, and nitrates and were monitored by ECG. Which patient most likely experienced myocardial infarction?

Selecciona una de las siguientes respuestas posibles:

  • A 67-year-old female whose pain started at 2 AM while she was asleep and responded to nitrates; the ECG showed arrhythmias and ST-segment elevation; cardiac markers remained stable.

  • An 80-year-old woman whose pain started at 6 AM shortly after awakening and was not relieved by nitrates or rest; the ECG showed ST-segment elevation; levels of cardiac markers subsequently rose.

  • A 33-year-old male whose pain started at 7 AM during moderate exercise and was relieved by nitrates; ECG was normal; cardiac markers remained stable.

  • A 61-year-old man whose pain started at 9 AM during a short walk and responded to nitrates, but not to rest; ECG and cardiac markers remained stable, but anginal pattern worsened.

Explicación

Pregunta 37 de 130

1

Cigarette smoking is a risk factor in coronary artery disease because smoking:

Selecciona una de las siguientes respuestas posibles:

  • promotes platelet adhesion.

  • increases serum HDL levels.

  • decreases serum lipid levels.

  • reduces vasoconstriction and peripheral resistance.

Explicación

Pregunta 38 de 130

1

When comparing angina with myocardial infarction (MI), which statement is true?

Selecciona una de las siguientes respuestas posibles:

  • Pain is more severe and lasts longer with angina than with MI.

  • Both angina and MI cause tissue necrosis.

  • Angina pain is relieved by rest and intake of nitroglycerin; the pain of MI is not.

  • Angina often occurs at rest; MI occurs during a stressful time.

Explicación

Pregunta 39 de 130

1

Which of the following hypertensive individuals is most likely to have his or her high blood pressure diagnosed as secondary rather than essential?

Selecciona una de las siguientes respuestas posibles:

  • A 40-year-old smoker who eats excessive amounts of salt and saturated fats

  • An African American man who leads a sedentary lifestyle

  • A 69-year-old woman who is an executive in a large corporation

  • A 51-year-old male who has been diagnosed with glomerulonephritis

Explicación

Pregunta 40 de 130

1

A 54-year-old man with a long-standing diagnosis of essential hypertension is meeting with his physician. The patient's physician would anticipate that which of the following phenomena is most likely occurring?

Selecciona una de las siguientes respuestas posibles:

  • The conversion of angiotensin I to angiotensin II in his lungs causes increases in blood pressure and sodium reabsorption.

  • The patient's juxtaglomerular cells are releasing aldosterone as a result of sympathetic stimulation.

  • ADH is exerting an effect on his chemoreceptors and baroreceptors resulting in vasoconstriction.

  • Adrenalin from his adrenal gland is initiating the renin -angiotensin- aldosterone system.

Explicación

Pregunta 41 de 130

1

What condition refers to thrombotic events associated with venous wall inflammation, and occurs in any part of the body?

Selecciona una de las siguientes respuestas posibles:

  • phlebothrombosis

  • deep vein thrombosis

  • varicose veins

  • thrombophlebitis

Explicación

Pregunta 42 de 130

1

Which vessel/s is/are particularly prone to aneurysm?

Selecciona una de las siguientes respuestas posibles:

  • aorta

  • superior vena cava

  • small peripheral arteries of the legs

  • inferior vena cava

Explicación

Pregunta 43 de 130

1

A drug taken in small doses on a continuing basis to reduce platelet adhesion is:

Selecciona una de las siguientes respuestas posibles:

  • Aspirin

  • Streptokinase

  • Heparin

  • Ibuprofen

Explicación

Pregunta 44 de 130

1

A venous disorder in which a thrombus forms spontaneously in a vein without prior inflammation is referred to as:

Selecciona una de las siguientes respuestas posibles:

  • Venous stenosis

  • Thrombophlebitis

  • Fibrothrombosis

  • Phlebothrombosis

Explicación

Pregunta 45 de 130

1

A general term for all types of arterial changes, but most often applied to loss of elasticity of arteries and arterioles, is:

Selecciona una de las siguientes respuestas posibles:

  • Vascular stenosis

  • Atherosclerosis

  • Arteriosclerosis

  • Osteoporosis

Explicación

Pregunta 46 de 130

1

A modifiable factor that increases the risk for atherosclerosis is:

Selecciona una de las siguientes respuestas posibles:

  • leading a sedentary lifestyle.

  • being female and older than 40 years of age.

  • familial hypercholesterolemia.

  • excluding saturated fats from the diet.

Explicación

Pregunta 47 de 130

1

A coronary artery disease that occurs when the blood flow to the myocardial cells is interrupted for an extended period of time, followed by necrosis, is referred to as:

Selecciona una de las siguientes respuestas posibles:

  • Angina pectoris

  • Atherosclerosis

  • Myocardial infarction

  • Dysrhythmias

Explicación

Pregunta 48 de 130

1

Which antihypertensive drug group should be avoided when a person has a pre-existing respiratory condition like asthma?

Selecciona una de las siguientes respuestas posibles:

  • Calcium channel antagonists.

  • Diuretics.

  • β-blockers.

  • ACE inhibitors.

Explicación

Pregunta 49 de 130

1

Adverse effects such as an unproductive cough sometimes occur with which category of medicine frequently used for treatment of hypertension?

Selecciona una de las siguientes respuestas posibles:

  • Calcium channel blockers

  • Diuretics

  • β-blockers

  • ACE inhibitors

Explicación

Pregunta 50 de 130

1

Factors that may lead to the formation of varicose veins include:

Selecciona una de las siguientes respuestas posibles:

  • Trauma, lying in bed for prolonged periods, and liver failure

  • Pregnancy, vitamin deficiencies, and mitral valve defects

  • Vein valve damage, wearing tight clothing, and crossing legs

  • Diet, exercise, and obesity

Explicación

Pregunta 51 de 130

1

How do ACE inhibitors work?

Selecciona una de las siguientes respuestas posibles:

  • They block Angiotensinogen production in the lungs

  • They block the enzyme cyclooxygenase

  • They block the formation of the enzyme renin by the kidney

  • They block the conversion of Angiotension 1 to Angiotensin 2

Explicación

Pregunta 52 de 130

1

A 13-year-old boy has had a sore throat for at least a week and has been vomiting for 2 days. His glands are swollen, and he moves stiffly because his joints hurt. Throat cultures show infection with group A streptococci. This child is at high risk for?

Selecciona una de las siguientes respuestas posibles:

  • mitral valve stenosis.

  • vasculitis.

  • myocarditis.

  • infective endocarditis.

Explicación

Pregunta 53 de 130

1

A client has been diagnosed with mitral valve stenosis following his recovery from rheumatic fever. Which of the following teaching points would be most accurate to convey to the client?

Selecciona una de las siguientes respuestas posibles:

  • Your heart's mitral valve isn't closing properly so blood is flowing backward in your heart and eventually into your lungs.

  • The valve between your left ventricle and left atria is infected and isn't allowing enough blood through.

  • The normal tissue that makes up the valve between the right sides of your heart has stiffened.

  • Your mitral valve isn't opening up enough for blood to flow into the part of your heart that sends blood into circulation.

Explicación

Pregunta 54 de 130

1

On a routine physical exam visit, the physician mentions that he hears a new murmur. The patient gets worried and asks, “What does this mean?” The physician responds;

Selecciona una de las siguientes respuestas posibles:

  • One of your heart valves is not opening properly. We need to do an echocardiogram to see which valve is having problem.

  • This may make you a little more fatigued than usual. Let me know if you start getting dizzy or light-headed.

  • This could be caused by an infection. Have you been feeling well the past few weeks?

  • It would be caused by stress. Let's keep our eye on it and see if it goes away with your next visit.

Explicación

Pregunta 55 de 130

1

A 34-year-old man who is an intravenous drug user has presented to the emergency department with malaise, abdominal pain, and lethargy. The health care team wants to rule out endocarditis as a diagnosis. Staff of the department would most realistically anticipate which of the following sets of diagnostics?

Selecciona una de las siguientes respuestas posibles:

  • CT of the heart, chest x-ray, and ECG

  • ECG, blood pressure, and stress test

  • Echocardiogram, blood cultures, and temperature

  • Cardiac catheterization, chest x-ray, electrolyte measurement, and white cell count

Explicación

Pregunta 56 de 130

1

A physician has ordered the measurement of a cardiac patient's electrolyte levels as part of the client's morning blood work. Which of the following statements best captures the importance of potassium in the normal electrical function of the patient's heart?

Selecciona una de las siguientes respuestas posibles:

  • Potassium catalyzes the metabolism of ATP, producing the gradient that results in electrical stimulation.

  • Potassium is central to establishing and maintaining the resting membrane potential of cardiac muscle cells.

  • The reciprocal movement of one potassium ion for one sodium ion across the cell membrane results in the production of an action potential.

  • The impermeability of cardiac cell membranes to potassium allows for action potentials achieved by the flow of sodium ions.

Explicación

Pregunta 57 de 130

1

The initial medical management for a symptomatic patient with obstructive hypertrophic cardiomyopathy (HCM) would be administering a medication to block the effects of catecholamines. The nurse will anticipate administering which of the following medications?

Selecciona una de las siguientes respuestas posibles:

  • Propranolol, a beta-adrenergic blocker

  • Lanoxin, an inotropic

  • Lisinopril, an ACE inhibitor

  • Lasix, a diuretic

Explicación

Pregunta 58 de 130

1

A 63-year-old male client has been diagnosed with a bundle branch block. How will this client's care team most likely expect his condition to be expressed diagnostically?

Selecciona una de las siguientes respuestas posibles:

  • His ECG will show a flattened P wave as a result of impaired atrial depolarization.

  • His ECG will show an inordinately wide QRS complex because impulses are bypassing the normal conduction tissue.

  • Conduction from the Purkinje fibers to the bundle branches is compromised by inadequate conduction.

  • His AV node will be performing the primary pacemaker role due to inadequacy of the SA node.

Explicación

Pregunta 59 de 130

1

You are looking after a 61-year-old male client in the hospital who has presented with a new onset of atrial fibrillation. Which of the following courses of treatment would you most likely expect the attending physician to initiate?

Selecciona una de las siguientes respuestas posibles:

  • Diuretics, total bed rest, and cardioversion if necessary

  • Anticoagulants and beta-blockers to control rate

  • Antihypertensives and constant cardiac monitoring in a high acuity unit

  • Immediate cardioversion followed by surgery to correct the atrial defect

Explicación

Pregunta 60 de 130

1

The amount of blood pumped by one ventricle in one minute, is called the

Selecciona una de las siguientes respuestas posibles:

  • stroke volume.

  • cardiac output.

  • ejection fraction.

  • end-diastolic volume.

Explicación

Pregunta 61 de 130

1

If someone's heart has a stroke volume of 70 ml and a heart rate of 90 beat/minute, the cardiac output would be

Selecciona una de las siguientes respuestas posibles:

  • 70 ml/min

  • 6.3 L/min

  • 1.28 ml/min

  • 0.77 L/min

Explicación

Pregunta 62 de 130

1

Any mechanism that increases heart rate is said to have a positive ____ effect.

Selecciona una de las siguientes respuestas posibles:

  • chronotropic

  • inotropic

  • cholinergic

  • feedback

Explicación

Pregunta 63 de 130

1

Stroke volume is regulated by all of the following except

Selecciona una de las siguientes respuestas posibles:

  • end-diastolic volume

  • peripheral resistance

  • cardiac output

  • contractility

Explicación

Pregunta 64 de 130

1

The preload acting on a ventricle is equivalent to that chamber's

Selecciona una de las siguientes respuestas posibles:

  • end-diastolic volume

  • stroke volume

  • contractility

  • ejection fraction

Explicación

Pregunta 65 de 130

1

The afterload imposed on a ventricle refers to

Selecciona una de las siguientes respuestas posibles:

  • the amount of blood added to a ventricle by atrial systole

  • the total peripheral resistance opposing the ejection of blood

  • its end-systolic volume, the blood left after contraction is complete

  • the ejection fraction, or percentage of EDV ejected by ventricular systole

Explicación

Pregunta 66 de 130

1

The Frank-Starling law of the heart describes the proportional relationship between

Selecciona una de las siguientes respuestas posibles:

  • stroke volume and end-diastolic volume

  • stroke volume and cardiac output

  • stroke volume is and total peripheral resistance

  • the left and right ventricles

Explicación

Pregunta 67 de 130

1

A positive inotropic agent is something that

Selecciona una de las siguientes respuestas posibles:

  • decreases the contractility of myocardial fibers

  • increases the contractility of myocardial fibers

  • reduces the heart rate in positive feedback loop

  • increases the heart rate in positive feedback loop

Explicación

Pregunta 68 de 130

1

A nurse is administering morning medications to a number of patients on a medical unit. Which of the following medication regimens is most suggestive that the patient has a diagnosis of heart failure?

Selecciona una de las siguientes respuestas posibles:

  • Anticoagulant, antihypertensive, calcium supplement

  • Beta-blocker, potassium supplement, anticoagulant

  • Diuretic, ACE inhibitor, beta-blocker

  • Antihypertensive, diuretic, antiplatelet aggregator

Explicación

Pregunta 69 de 130

1

A nurse will be providing care for a female patient who has a diagnosis of heart failure that has been characterized as being primarily right sided. Which of the following statements best describes the presentation that the nurse should anticipate? The client

Selecciona una de las siguientes respuestas posibles:

  • has cyanotic lips and extremities, low urine output, and low blood pressure.

  • has pitting edema to the ankles and feet bilaterally, decreased activity tolerance, and occasional upper right quadrant pain.

  • complains of dyspnoea and has adventitious breath sounds on auscultation (listening).

  • has a distended bladder, facial edema, and nighttime difficulty breathing.

Explicación

Pregunta 70 de 130

1

A 3-year-old child with right-sided heart failure has been admitted for worsening of his condition. Which of the following assessments would be considered one of the earliest signs of systemic venous congestion in this toddler?

Selecciona una de las siguientes respuestas posibles:

  • Breathlessness with activity

  • Increased urine output

  • Enlargement of the liver

  • Excessive crying

Explicación

Pregunta 71 de 130

1

The nurse working in the ICU knows that chronic elevation of left ventricular end-diastolic pressure will result in the patient displaying which of the following clinical manifestations?

Selecciona una de las siguientes respuestas posibles:

  • Chest pain and intermittent ventricular tachycardia

  • Dyspnoea and crackles in bilateral lung bases

  • Petechia and spontaneous bleeding

  • Muscle cramping and cyanosis in the feet

Explicación

Pregunta 72 de 130

1

Which one of the following is not a pathophysiological change associated with heart failure?

Selecciona una de las siguientes respuestas posibles:

  • Decreased angiotensin II production.

  • Decreased myocardial contractility.

  • Increased myocardial oxygen demand.

  • Cardiac remodelling.

Explicación

Pregunta 73 de 130

1

Pulmonary symptoms, such as dyspnoea and cough, common to left heart failure are a result of:

Selecciona una de las siguientes respuestas posibles:

  • pulmonary vascular congestion.

  • hypoxaemia.

  • inflammatory pulmonary oedema.

  • bronchoconstriction.

Explicación

Pregunta 74 de 130

1

Adverse effects such as an unproductive cough and taste disturbances may occur from which category of medicine frequently used for treatment of heart failure?

Selecciona una de las siguientes respuestas posibles:

  • Aldosterone antagonists.

  • Cardiac glycosides.

  • ACE inhibitors.

  • Diuretics.

Explicación

Pregunta 75 de 130

1

Right heart failure will likely cause:

Selecciona una de las siguientes respuestas posibles:

  • non-pitting oedema in the arms, resulting from decreased arterial pressure

  • pitting oedema in the lower legs, resulting from increased venous pressure

  • pulmonary oedema from increased left ventricular filling

  • all answers are correct

Explicación

Pregunta 76 de 130

1

A 65-year-old male is diagnosed with chronic pulmonary disease and elevated pulmonary vascular resistance. Which of the following heart failures generally results from this condition?

Selecciona una de las siguientes respuestas posibles:

  • low-output failure

  • right heart failure

  • high-output failure

  • left heart failure

Explicación

Pregunta 77 de 130

1

Excess preload can be reduced by:

Selecciona una de las siguientes respuestas posibles:

  • use of antidiuretics

  • use of diuretics

  • increasing volume intake, that is, drinking more fluid

  • all answers are correct

Explicación

Pregunta 78 de 130

1

In terms of cardiac pathology, the greatest danger of untreated rheumatic fever is:

Selecciona una de las siguientes respuestas posibles:

  • myocardial hypertrophy

  • damage to heart valves

  • atherosclerosis

  • acute right-side heart failure

Explicación

Pregunta 79 de 130

1

An incompetent mitral valve would cause:

Selecciona una de las siguientes respuestas posibles:

  • decreased pressure in the left atrium.

  • decreased output from the left ventricle.

  • hypertrophy of the right ventricle.

  • increased blood to remain in the right atrium.

Explicación

Pregunta 80 de 130

1

A very rapid heart rate reduces cardiac output because:

Selecciona una de las siguientes respuestas posibles:

  • conduction through the AV node is impaired.

  • ventricular filling is reduced.

  • venous return is increased.

  • ventricular fibrillation develops immediately.

Explicación

Pregunta 81 de 130

1

Pericarditis causes a reduction in cardiac output as a result of which of the following?

Selecciona una de las siguientes respuestas posibles:

  • Delays in the conduction system, interfering with cardiac rhythm

  • Excess fluid in the pericardial cavity, which decreases ventricular filling

  • Weak myocardial contractions due to friction rub

  • Incompetent valves, which allow regurgitation of blood

Explicación

Pregunta 82 de 130

1

A patient, who is experiencing some angina associated with tachycardia, has been placed on verapamil, a calcium channel blocker. Knowing the mechanism of action of this medication, you should assess this patient for which of the following adverse reactions?

Selecciona una de las siguientes respuestas posibles:

  • Ventricular tachycardia

  • Bradycardia

  • Increased cardiac output

  • Sudden asystole

Explicación

Pregunta 83 de 130

1

Which of the following drugs improves cardiac efficiency by slowing the heart rate and increasing the force of cardiac contractions?

Selecciona una de las siguientes respuestas posibles:

  • Epinephrine

  • Nifedipine

  • Digoxin

  • Furosemide

Explicación

Pregunta 84 de 130

1

Heart block, in which a conduction delay at the AV node results in intermittent missed ventricular contractions, is called:

Selecciona una de las siguientes respuestas posibles:

  • total heart block.

  • first-degree block.

  • second-degree block.

  • bundle-branch block.

Explicación

Pregunta 85 de 130

1

Atrial fibrillation may:

Selecciona una de las siguientes respuestas posibles:

  • increase the risk of thromboembolism

  • induce an irregular heart rate

  • increase turbulence within the atrial chamber

  • all answers are correct

Explicación

Pregunta 86 de 130

1

The current optimal drug therapy in the management of heart failure is:

Selecciona una de las siguientes respuestas posibles:

  • a non-selective β-blocker.

  • the dopamine agonist, dobutamine.

  • the cardiac glycoside, digoxin.

  • an ACE inhibitor and a diuretic.

Explicación

Pregunta 87 de 130

1

Which one of the following would you not expect to be administered to a patient suffering heart failure?

Selecciona una de las siguientes respuestas posibles:

  • diuretic

  • digoxin

  • beta-agonist

  • ACE inhibitor

Explicación

Pregunta 88 de 130

1

A 67-year-old female has chronic gastrointestinal bleeding. The primary cause of her anaemia is:

Selecciona una de las siguientes respuestas posibles:

  • bone marrow failure

  • folate deficiency

  • vitamin B12 deficiency

  • iron deficiency

Explicación

Pregunta 89 de 130

1

Maternal-fetal blood incompatibility may exist in which of the following conditions?

Selecciona una de las siguientes respuestas posibles:

  • The mother is Rh-negative and the fetus is Rh-positive.

  • The mother has type AB blood and the fetus has type B blood.

  • The mother has type A blood and the fetus has type O blood.

  • The mother is Rh-positive and the fetus is Rh-negative.

Explicación

Pregunta 90 de 130

1

A 5-year-old male is diagnosed with normocytic-normochromic anaemia. Which of the following anaemias falls into this category?

Selecciona una de las siguientes respuestas posibles:

  • thalassaemia

  • haemolytic anaemia

  • pernicious anaemia

  • iron deficiency anaemia

Explicación

Pregunta 91 de 130

1

A 35-year-old female is diagnosed with vitamin B12 deficiency anaemia. A decrease in which of the following is the most likely cause?

Selecciona una de las siguientes respuestas posibles:

  • transferin

  • intrinsic factor

  • gastric enzymes

  • ferritin

Explicación

Pregunta 92 de 130

1

Erythrocytes that are _____ contain an abnormally low concentration of haemoglobin.

Selecciona una de las siguientes respuestas posibles:

  • macrocytic

  • hypochromic

  • microcytic

  • hyperchromic

Explicación

Pregunta 93 de 130

1

Which of the following is typically not associated with anaemia?

Selecciona una de las siguientes respuestas posibles:

  • increased haemolysis

  • lack of intrinsic factor

  • high dietary iron intake

  • disruption of haemoglobin chains

Explicación

Pregunta 94 de 130

1

Those who live at high altitudes, or who receive extra erythropoietin, are likely to suffer:

Selecciona una de las siguientes respuestas posibles:

  • primary polycythaemia

  • haemolytic disease

  • secondary polycythaemia

  • sickle cell anaemia

Explicación

Pregunta 95 de 130

1

Anaemia due to inherited mutations that reduce the production of either alpha or beta haemoglobin chains is known as:

Selecciona una de las siguientes respuestas posibles:

  • haemolytic anaemia

  • thalassaemia

  • pernicious anaemia

  • all answers are correct

Explicación

Pregunta 96 de 130

1

Which blood disorder results from an autoimmune attack on parietal cells of the stomach wall?

Selecciona una de las siguientes respuestas posibles:

  • haemolytic disease of the newborn

  • pernicious anaemia

  • microcytic anaemia

  • thalassaemia

Explicación

Pregunta 97 de 130

1

What medical term is used to describe a marked reduction in platelets?

Selecciona una de las siguientes respuestas posibles:

  • haemorrhoids

  • haemophilia

  • thrombocytopenia

  • polycythaemia

Explicación

Pregunta 98 de 130

1

Multiple opportunistic infections develop with acute leukemia primarily because:

Selecciona una de las siguientes respuestas posibles:

  • many circulating leukocytes are immature

  • severe anemia interferes with the immune response

  • the number of white blood cells is decreased

  • decreased appetite and nutritional intake reduce natural defences

Explicación

Pregunta 99 de 130

1

The cause of leukaemia is unknown, but risk factors include ...

Selecciona una de las siguientes respuestas posibles:

  • Exposure to high amounts of radiation

  • Genetic disorders such as Down's syndrome

  • Exposure to carcinogens such as benzene

  • All answers are correct

Explicación

Pregunta 100 de 130

1

Why is excessive bleeding a common occurrence with acute leukaemia?

Selecciona una de las siguientes respuestas posibles:

  • Deficit of calcium ions

  • Dysfunctional thrombocytes

  • Impaired production of prothrombin and fibrinogen

  • Decreased platelets

Explicación

Pregunta 101 de 130

1

A 5-year-old female is diagnosed with acute leukaemia. She will most likely be treated with:

Selecciona una de las siguientes respuestas posibles:

  • bone marrow transplant

  • immunotherapy

  • localised radiation therapy

  • chemotherapy

Explicación

Pregunta 102 de 130

1

The Reed-Sternberg cell is diagnostic for:

Selecciona una de las siguientes respuestas posibles:

  • chronic myeloblastic leukemia.

  • multiple myeloma

  • non-Hodgkin’s lymphoma

  • Hodgkin’s lymphoma

Explicación

Pregunta 103 de 130

1

Which of the following applies to the leukaemia’s?

Selecciona una de las siguientes respuestas posibles:

  • Exposure to chemicals is not considered a predisposing factor.

  • Lymphoid tissue produces abnormal leukocytes.

  • Chronic leukemias are more common in older people.

  • All answers are correct.

Explicación

Pregunta 104 de 130

1

Predisposing factors to leukemia commonly include:

Selecciona una de las siguientes respuestas posibles:

  • exposure to radiation.

  • certain fungal and protozoal infections.

  • familial tendency.

  • cigarette smoking.

Explicación

Pregunta 105 de 130

1

What is the primary treatment for the leukemias?

Selecciona una de las siguientes respuestas posibles:

  • Chemotherapy

  • Radiation

  • Surgery

  • Immunotherapy

Explicación

Pregunta 106 de 130

1

Multiple myeloma is a malignant tumor involving:

Selecciona una de las siguientes respuestas posibles:

  • bone cells.

  • lymph nodes.

  • plasma cells.

  • granulocytes.

Explicación

Pregunta 107 de 130

1

In cases of polycythemia vera, blood pressure is most likely elevated as a result of:

Selecciona una de las siguientes respuestas posibles:

  • increased renin and aldosterone secretions.

  • increased blood volume/viscosity

  • congested spleen and bone marrow.

  • frequent infarcts in the coronary circulation

Explicación

Pregunta 108 de 130

1

Vitamin K is required by the liver to synthesize:

Selecciona una de las siguientes respuestas posibles:

  • bilirubin

  • amino acids

  • prothrombin

  • heparin

Explicación

Pregunta 109 de 130

1

_____________________ would result from a reduced number of RBCs in the blood?

Selecciona una de las siguientes respuestas posibles:

  • Decreased haematocrit

  • Increased haemoglobin count

  • Decreased osmotic pressure of the blood

  • Increased risk of haemostasis

Explicación

Pregunta 110 de 130

1

Haemolytic disease of the newborn is due to the presence of ________ and ________.

Selecciona una de las siguientes respuestas posibles:

  • Rh+ foetal blood, Anti-Rh antibodies in maternal blood

  • Rh+ foetal blood, Rh+ maternal blood

  • Rh- foetal blood, Rh+ maternal blood

  • Anti-Rh antibodies in foetal blood, lack of antibodies in maternal blood

Explicación

Pregunta 111 de 130

1

Lymphomas describe white blood cell malignancies that:

Selecciona una de las siguientes respuestas posibles:

  • do not involve bone marrow tissue initially

  • cause marked proliferation of white blood cells

  • initiate as cancers of lymphoid cells

  • all answers are correct

Explicación

Pregunta 112 de 130

1

Malignant neoplasms involving lymphocyte proliferation in the lymph nodes are called:

Selecciona una de las siguientes respuestas posibles:

  • lymphomas

  • leukaemias

  • myelomas

  • lymphocytomas

Explicación

Pregunta 113 de 130

1

Which of the following substances acts as an anticoagulant?

Selecciona una de las siguientes respuestas posibles:

  • Vitamin K

  • Prothrombin

  • Heparin

  • Fibrinogen

Explicación

Pregunta 114 de 130

1

A 67-year-old female is admitted to the ED with a diagnosis of polycythaemia vera. This has occurred as a result of:

Selecciona una de las siguientes respuestas posibles:

  • dehydration.

  • blood doping.

  • exposure to high altitude.

  • abnormal proliferation of red cells.

Explicación

Pregunta 115 de 130

1

In disseminated intravascular coagulation (DIC), active bleeding occurs after intravascular clotting because:

Selecciona una de las siguientes respuestas posibles:

  • clotting factors are depleted.

  • tissue factor (TF) is inactivated.

  • prothrombin is activated.

  • inflammatory mediators are released.

Explicación

Pregunta 116 de 130

1

________ describes a condition where malignant white cells produce vast quantities of abnormal immunoglobulins that ultimately destroy bone.

Selecciona una de las siguientes respuestas posibles:

  • Multiple myeloma

  • Chronic lymphocytic leukaemia

  • Agranulocytosis

  • Non-Hodgkin lymphoma

Explicación

Pregunta 117 de 130

1

A neoplastic disorder that involves malignant plasma cells that replace the bone marrow and erodes bone is referred to as:

Selecciona una de las siguientes respuestas posibles:

  • Aplastic anemia

  • Multiple myeloma

  • Leukaemia

  • Non-Hodgkin lymphoma

Explicación

Pregunta 118 de 130

1

What are the typical early clinical manifestations of anaemia?

Selecciona una de las siguientes respuestas posibles:

  • Jaundice, cyanosis

  • Pallor, dyspnea, and fatigue

  • Chest pain, palpitations

  • Bradycardia, cyanosis

Explicación

Pregunta 119 de 130

1

Which of the following individuals is at highest risk for developing a vitamin B12 deficiency anaemia?

Selecciona una de las siguientes respuestas posibles:

  • a 47-year-old male who had a gastrectomy procedure (removal of the stomach)

  • a 3-year-old female who is a fussy eater

  • a 64-year-old male with a history of duodenal ulcers and gastrointestinal bleeding

  • a 26-year-old female in the second trimester of her first pregnancy

Explicación

Pregunta 120 de 130

1

A diverse group of neoplasms developing from the proliferation of malignant lymphocytes in the lymphoid system is referred to as:

Selecciona una de las siguientes respuestas posibles:

  • Leukaemias

  • Microcytic anaemias

  • Lymphomas

  • Lymphatic anaemias

Explicación

Pregunta 121 de 130

1

In which of the following individuals would a clinician most suspect multiple myeloma as a diagnosis?

Selecciona una de las siguientes respuestas posibles:

  • A 70-year-old woman whose blood work reveals large numbers of immature granulocytes

  • An 81-year-old male resident of a long-term care home who has an uncommon bacterial pneumonia and who is unable to produce a fever

  • A 40-year-old man who has had three broken bones over the past 6 months and whose serum calcium and creatinine levels are elevated

  • A 68-year-old former coal miner who has white cell levels exponentially higher than normal ranges

Explicación

Pregunta 122 de 130

1

A 34-year-old male presents in the ED with extreme fatigue and shortness of breath. His skin and sclera appear to have a yellowish discoloration. These findings are consistent with which type of anaemia?

Selecciona una de las siguientes respuestas posibles:

  • pernicious anaemia

  • aplastic anaemia

  • iron deficiency anaemia

  • haemolytic anaemia

Explicación

Pregunta 123 de 130

1

A 40-year-old female develops disseminated intravascular coagulation (DIC). The most likely cause of this condition is:

Selecciona una de las siguientes respuestas posibles:

  • lack of vitamin B12.

  • blood transfusion.

  • snake venom.

  • sepsis.

Explicación

Pregunta 124 de 130

1

A 52-year-old male IV drug user 5years ago was diagnosed with hepatitis C. He is now experiencing impaired blood clotting. A decrease in which of the following vitamins is most likely the cause?

Selecciona una de las siguientes respuestas posibles:

  • E

  • K

  • B12

  • D

Explicación

Pregunta 125 de 130

1

Chronic blood loss causes anaemia because of the:

Selecciona una de las siguientes respuestas posibles:

  • lower metabolic rate

  • shortened life span of the erythrocytes

  • loss of protein and electrolytes

  • smaller amount of recycled iron available

Explicación

Pregunta 126 de 130

1

What is the basic abnormality in thalassemia?

Selecciona una de las siguientes respuestas posibles:

  • Several amino acids in the Haem chains have been replaced by substitute amino acids.

  • The iron molecule is displaced in haemoglobin.

  • More than four globin chains are found in the erythrocytes.

  • There is failure to synthesize either the alpha or beta chains in the haemoglobin molecule.

Explicación

Pregunta 127 de 130

1

A 60-year-old woman is suspected of having non-Hodgkin lymphoma (NHL). Which of the following aspects of her condition would help to rule out Hodgkin lymphoma?

Selecciona una de las siguientes respuestas posibles:

  • The woman complains of recent debilitating fatigue.

  • Her neoplasm originates in secondary lymphoid structures.

  • The lymph nodes involved are located in a large number of locations in the lymphatic system.

  • The presence of Reed-Sternberg cells has been confirmed.

Explicación

Pregunta 128 de 130

1

One of the reasons non-Hodgkin’s lymphomas are harder to treat than Hodgkin’s lymphomas is that they

Selecciona una de las siguientes respuestas posibles:

  • are not affected by the newer drug treatments.

  • are asymptomatic until they reach stage IV.

  • tend to be much larger than Hodgkin’s lymphomas.

  • involve multiple nodes and widespread metastases.

Explicación

Pregunta 129 de 130

1

Individuals with liver disease often suffer from coagulation disorders because:

Selecciona una de las siguientes respuestas posibles:

  • treatment medications for liver failure cause fibrinolysis.

  • the liver is often the site of platelet pooling.

  • high levels of bilirubin interfere with the clotting system.

  • clotting factors are produced in the liver.

Explicación

Pregunta 130 de 130

1

In individuals with acute leukaemia, the increased number of malignant leukocytes leads to:
1. decreased haemoglobin.
2. thrombocytopenia.
3. bone pain only with increased activity.
4. splenomegaly.

Selecciona una de las siguientes respuestas posibles:

  • 1,3

  • 1,2,4

  • 1,2,3,4

  • 2,3,4

Explicación